New answers tagged

1 vote
Accepted

Let $\mu : [0, T] \to \mathcal P_2^a (\mathbb R^d), t \mapsto \mu_t$ be absolutely continuous. Is $t \mapsto \mathcal H (\mu_t)$ continuous?

$\newcommand{\R}{\mathbb R}$The answer is NO. I will provide below a counterexample in dimension $d=1$. Preliminaries: Let's agree that the entropy is $$ H(\rho)=\int_{\mathbb R}\rho(x)\log\rho(x) dx ...
leo monsaingeon's user avatar

Top 50 recent answers are included